find the value of x

Find The Value Of X

Answers

Answer 1

[tex]\textit{using the pythagorean theorem} \\\\ c^2=a^2+b^2 \qquad \begin{cases} c=\stackrel{hypotenuse}{11 + x}\\ a=\stackrel{adjacent}{x}\\ b=\stackrel{opposite}{17}\\ \end{cases}\implies (11+x)^2=x^2+17^2 \\\\\\ (11+x)(11+x)=x^2+17^2\implies \stackrel{F~O~I~L}{121+22x+x^2}~~ = ~~x^2+289 \\\\\\ 121+22x=289\implies 22x=168\implies x=\cfrac{168}{22}\implies x=\cfrac{84}{11}\implies x=7\frac{7}{11}[/tex]


Related Questions

Fine the area of the circle. Round to the nearest tenth. Use 3.14 for pie.

Answers

Answer:

the answer is 379.9m²

Step-by-step explanation:

half of 22 is ll time 3.14 equals 379.94. to the nearest tenth is 379.9

choose the name of the highlighted part.​

Answers

Answer:

B.edge it is the correct answer!

calculate the arithmetic mean of -121 and 49

Answers

Answer:

i dont rlly know

Step-by-step explanation:

-121+49 then divide by 2

Answer:

  -36

Step-by-step explanation:

The arithmetic mean of n numbers is their sum, divided by n.

The arithmetic mean of your 2 numbers is ...

  (-121 +49)/2 = -72/2 = -36

The arithmetic mean of -121 and 49 is -36.

__

Many calculators and spreadsheets have an arithmetic mean function built in.

Helpppppp!!!
(Show work)

Answers

Answer:

Step-by-step explanation:

Part B: A"C

You have just inherited $150,261 from a trust that has matured. You would like to invest the total amount into an account that pays you an annual compounded interest rate of 9. 1%, and you would like to make annual withdrawals over the next 20 years such that by the end of this 20 year period, the amount remaining in the account will be zero dollars. Determine, from the given information, the amount of the annual withdrawals. Round to the nearest cent. A. $16,578. 03 c. $15,978. 10 b. $162,851. 31 d. $15,575. 98.

Answers

Answer:

C

Step-by-step

The amount of the annual withdrawals will be $16,574.24.

The given data in the problem is;

P = annual payment

PV= present value = $150,261

r= rate of intreast = 9.1% =9.1/100= 0.091

n = time period= 20

What is the formula for the annual payment ?

[tex]P=\frac{r\times PV}{1-(1+r)^{-n}}[/tex]

[tex]P=\frac{0.091\times 150261}{1-(1+0.091)^{-20}}[/tex]

[tex]P=16754.24[/tex]

Therefore, the amount of the annual withdrawals will be $16,574.24.

To learn more about the amount visit:

https://brainly.com/question/25870256

Please help me with this math question. I will give brainliest!! :D

Answers

Answer:

C

Step-by-step explanation:

Nathan is going to invest in an account paying an interest rate of 4. 9% compounded continuously. How much would Nathan need to invest, , for the value of the account to reach $34,000 in 9 years?

Answers

[tex]~~~~~~ \textit{Continuously Compounding Interest Earned Amount} \\\\ A=Pe^{rt}\qquad \begin{cases} A=\textit{accumulated amount}\dotfill & \$34000\\ P=\textit{original amount deposited}\\ r=rate\to 4.9\%\to \frac{4.9}{100}\dotfill &0.049\\ t=years\dotfill &9 \end{cases} \\\\\\ 34000=Pe^{0.049\cdot 9}\implies 34000=Pe^{0.441}\implies \cfrac{34000}{e^{0.441}}=P\implies 21875.35\approx P[/tex]

What is the simplified expression of (4-5r+8s)(5r-9)

Answers

Answer:

The simplified expression of (4-5r+8s)(5r-9) is

40r s-25r^2-72s+65r-36

(hope this helped)

The simplified expression of (4-5r+8s)(5r-9) is [tex]65r - 25r^2 + 9r + 40rs - 72s.[/tex]

The simplified expression of (4-5r+8s)(5r-9) step by step.

Understanding how to simplify expressions can be incredibly useful in algebra and will make solving equations much easier. By using the distributive property, we can multiply the terms inside the parentheses and then combine like terms to get the simplified expression.

To simplify the expression (4-5r+8s)(5r-9), we'll use the distributive property, also known as the FOIL method (First, Outer, Inner, Last). This property allows us to multiply each term in the first parentheses by each term in the second parentheses.

1. First, we multiply the term "4" in the first parentheses by each term in the second parentheses:

4 * 5r = 20r

4 * (-9) = -36

2. Next, we move to the second term in the first parentheses, "-5r," and multiply it by each term in the second parentheses:

-5r * 5r = -25r²

-5r * (-9) = 45r

3. Lastly, we move to the third term in the first parentheses, "8s," and multiply it by each term in the second parentheses:

8s * 5r = 40rs

8s * (-9) = -72s

Now, we have all the resulting terms:

20r - 36 - 25r² + 45r + 40rs - 72s.

To simplify further, we combine like terms. Like terms have the same variable(s) raised to the same power:

20r + 45r = 65r

-36 + 45r = 9r

So, the simplified expression is:

65r - 25r² + 9r + 40rs - 72s.

In mathematical terms, we applied the distributive property to multiply each term in the first parentheses by each term in the second parentheses. Then, we combined like terms to obtain the final simplified expression, 65r - 25r² + 9r + 40rs - 72s.

To know more about Algebra here

https://brainly.com/question/29074939

#SPJ2

This figure represents the base of an aquarium that is 9 inches high.

What is the volume of the aquarium?


558 in³

612 in³

702 in³

810 in³

Answers

Answer:

Step-by-step explanation:

Answer:

558 in³ for K12

At what point does the line given by the following equation cross the y-axis? y = (6/9)X + 9

A. (9,0)
B. (0,6/9)
C. (0,9/6)
D. (0,9)​

Answers

Answer:

D

Step-by-step explanation:

the equation of a line in slope- intercept form is

y = mx + c ( m is the slope and c the y- intercept )

y = [tex]\frac{6}{9}[/tex] x + 9 ← is in slope- intercept form

with y- intercept c = 9

then the line crosses the y- axis at (0, 9 )

Jay is measuring the angles of a triangle. He gets 30 degrees and 100 degrees, what is the measured third angle?

Answers

Answer:

50°

Step-by-step explanation:

the sum of the 3 angles in a triangle = 180° , that is

3rd + 30° + 100° = 180°

3rd + 130° = 180° ( subtract 130° from both sides )

3rd angle = 50°

Answer:

50 degrees

Step-by-step explanation:

In any triangle, there are 180 degrees. If we add 100 degrees plus 30 degrees to get 130, simply subtract that from 180 the get the measure of the third angle. (180-130= 50) Hope this helps!

I need help with this math please, thanks in advance. Much appreciated. 50 points available! The most helpful answer will receive brainliest!

1. Sofie decides to develop a survey.
(a) Give an example of a question she could ask on her survey.
(b) How could Sofie select a simple random sample of students to take her survey?
(c) She gives out 80 surveys but receives only 32 completed surveys. What are the sample and population for Sofie’s research?
(d) Of the 32 students who completed surveys, 16 said they use social media while doing schoolwork. If Sofie uses only the completed surveys, what conclusion could she make about the percent of all high school students who use social media while doing schoolwork?


2.Tony decides to use observation. He visits 10 students and observes them doing schoolwork at home for 30 min. If a student looks at social media during that time, he notes it.
(a) Provide an advantage of using observation.
(b) Provide a disadvantage of using observation.
(c) What are the sample and population for Tony’s research?
(d) Suppose 3 of the 10 students looked at social media during the observation. What conclusion could Tony make about the percent of high school students who use social media while doing schoolwork?

Much thanks and appreciation!

Answers

Answer:

1. A. how much of the students time is spent on social media during school hours.

B. By using some kind of selection technique Like Having students draw number from a hat, who ever have the specific numbers will take her survey.

C. The population for Sofies research would be a rate of 5/2. For every 5 people who take the survey only 2 turned it in.

D. She would conclude that 1 out of every 2 or 50 percent of students use social media while doing school work.

2. a) It helps to get accurate data or records of the information he wants to process and also is a direct way of carrying out analysis.

b) The students definitely might get distracted in the process of his observation and there fore render the data collection not as accurate as it will be for analysis.

c) The sample are the 10 students

  The  population are high school students

d) Tony 's conclusion will be 3 out of 10 looked at social media

thus 3/10 * 100 = 30 %

Step-by-step explanation:

Use mental math to find the sum.

150 + 20 + 25

Answers

Answer:


50 + 20 = 70
70 + 25 = 95
150 + 95 = 245

In conclusion, 150 + 20 + 25 equals to a total amount of 215.


Step-by-step explanation:

Have a great rest of your day
#TheWizzer

hemo

150 + 20 + 25

= 100 +50+20+20+5

= 100+90+5

=195

Chin-Mae has a fishtank that measures 75 cm by 45 cm by 35 cm. He also has a jug that holds 1.75 litres. He uses the jug to fill the fishtank with water. How many full jugs of water does it take to fill the fishtank? Show how to check your answer. ​

Answers

Key ConceptsConverting between cm³ and mL1 cm³ = 1 mLFinding the volume of rectangular prisms

To find the volume of a rectangular prism, we can use the following formula:

[tex]V=lwh[/tex]

l = lengthw = widthh = height

Converting between mL and L

1000 mL = 1 L

Solving the Question

We're given:

Fish tank measurements: 75 cm by 45 cm by 35 cmJug capacity = 1.75 L

To solve this question, we could follow these steps:

Find the volume of the tankDivide the volume of the tank by the jug capacity

Find the volume of the tank

[tex]V=lwh[/tex]

⇒ Plug in the measurements: 75 cm by 45 cm by 35 cm
[tex]V=75*45*35\\V=118125[/tex]

Therefore, the volume of the tank is 118125 cm³.

Divide the volume of the tank by the capacity of the jug

There is a relationship between cm³ and mL, not cm³ and L. Convert the 1.75 L for the jug capacity to mL:

1.75 × 1000

= 1750

118125 ÷ 1750

= 67.5

Answer

It takes 68 full jugs of water to fill the tank.

Ten tubs of dried cranberries cost $40 how many runs can u buy for $20

Answers

Answer:

5 tubs

Step-by-step explanation:

This because if we divided 10 in half we get 5 and if we divide 40 in half we get 20

I hope this helps :)!!  

e + 3.1 = 5

this is hard.

Answers

AnswerAnswer:E=1.9

Step-by-step explanation:

ssorry bit i conta speak englessh

The population of Bloom Falls, Mass (population 937)is slowly moving to a
bigger city. Every year the population drops by 4.5%.
What is the population after 3 years

Answers

Answer:

Step-by-step explanation:

When the population drops by 4.5%, there will be (1 - 4.5%) = 0.955 portion of the original population staying.  When it happens 3 years in a row with an original population of 937, there will be:

937*0.955*0.955*0.955 = 816.11

~816 people staying in Bloom Falls.

Rachel deposited $10 in an account earning 5% interest compounded annually.
To the nearest cent, how much interest will she earn in 3 years?

Answers

The amount that Rachel would have in her account after 3 years with annual compounding is $11.58.

What would be the worth of the account in 3 years?

When interest is compounded annually, it means both the amout deposited and the interest already accrued increases in value once a year.

The formula for calculating future value:

FV = P (1 + r)^n

FV = Future value P = Present value R = interest rate N = number of years

$10 x (1.05)^3 = $11.58

To learn more about future value, please check: https://brainly.com/question/18760477

Answer:

1.58

Step-by-step explanation:

its right

Find the area. Round to the nearest hundredth when necessary. Be sure you are showing all work to find each figure.

Answers

Additional requirements in figure :-

Mark point :-

ABCDE

Draw a straight line from E to AB .

And the point line joins mark it as "F" .

It will generate quadrilateral FBCD.

we get to know In quadrilateral left angle is 90°

How ?

{

proof :

As three angles are given 90° So third angle will also be 90°

Reason:

→ Sum of interior angles = 2 (no. of angles - 2 × 180°

→Sum of interior angles = (4-2 × 180°)

→Sum of interior angles = (2 × 180°)

→Sum of interior angles = 360°

}

✿Now let the left angle be x

90° +90° + 90° + x = 360° 180° + 90° + x = 360°270° + x = 360x = 360° - 270°x = 90°

we know :

Area of rectangle = Length × Breadth

STEPS :

Area of rectangle = Length × Breadth

Area of rectangle = 7 × 8

Area of rectangle = 56 in²

Now let's find EF :

EF = BD - EC

EF = 7 - 4

EF = 3 in

To find A

F :

A•F = AB - CD

A•F = 12 - 8

A•F = 4 in

In triangle AFE:

EF is base of triangle A•F is height of triangle

We know :

Area of triangle =( Height × Base)/2

Steps :

Area of triangle = (Height × Base)/2Area of triangle = (4 × 3)/2Area of triangle = 12/2Area of triangle = 6 in²

To find area of figure :

Area of figure = Area of rectangle + Area of triangleArea of figure = 56 + 6Area of figure = 62 in²

______________________

~WindyMint

Answer:

[tex]\displaystyle 62\:in.^2[/tex]

Step-by-step explanation:

▽ [tex]\displaystyle \frac{hb}{2} = A, \frac{1}{2}bh = A, or\:\frac{1}{2}hb = A[/tex]

▯ [tex]\displaystyle hb = A[/tex]

All edges meet at right angles, therefore resulting in this:

[tex]\displaystyle 62 = 56 + 6 \Rightarrow 62 = 7 \times 8 + 4 \times 1\frac{1}{2}[/tex]

I am joyous to assist you at any time.

Shana spends $18 on some almonds. She pays for the almonds with two $10 bills. How much change does Shana get back? Enter your answer in the box. $

Answers

The correct answer is $2

Answer:

$2

Step-by-step explanation:

2($10)=$20

$20-$18=$2

a shopkeeper selling an article at discount of 20% looses rs 400 if he allow 10% discount he gains rs 200 find the marked price and the cost price of the article​

Answers

Answer:700

Step-by-step explanation:

Let the M.P of the article be x .

A discount of 20% was given on the M.P .

So the selling price of the article ( S.P ) will be M.P - Discount .

Discount = 20 % of x

⇒ Discount = 20/100 of x

⇒ Discount = 1/5 × x

⇒ Discount = x/5

Now S.P = M.P - Discount .

⇒ S.P = x - x/5

⇒ S.P = ( 5 x - x ) / 5

⇒ S.P = 4 x / 5

The shopkeeper loses Rs 200 .

Loss = C.P - S.P

⇒ C.P = Loss + S.P

⇒ C.P = 200 + 4 x / 5

⇒ C.P = ( 1000 + 4 x )/5

Now if he allows 10 % discount then the gain is Rs 150 .

Gain = S.P - C.P

⇒ S.P = Gain + C.P

⇒ S.P = 150 + ( 1000 + 4 x )/5

⇒ S.P = ( 750 + 1000 + 4 x )/5

⇒ S.P = ( 1750 + 4 x )/5

Discount = 10 % of x .

⇒ Discount = 10/100 × x

⇒ Discount = x/10

S.P = M.P - Discount

⇒ ( 1750 + 4 x )/5 = x - x/10

⇒ ( 1750 + 4 x )/5 = 9 x / 10

⇒ 1750 + 4 x = 9 x / 2

⇒ 3500 + 4 x = 9 x

⇒ 9 x - 4 x = 3500

⇒ 5 x = 3500

⇒ x = 3500/5

⇒ x = 700

A parabola can be represented by the equation y2 = –x. What are the coordinates of the focus and the equation of the directrix? focus: (negative one-fourth, 0); directrix: x = One-fourth focus: (one-fourth, 0); directrix: x = Negative one-fourth focus: (–4,0); directrix: x = 4 focus: (4,0); directrix: x = –4.

Answers

The focus and directrix of the parabola are (–0.25, 0) and x = 0.25. Then the correct option is A.

What is the parabola?

It is the locus of a point that moves so that it is always the same distance from a non-movable point and a given line. The non-movable point is called focus and the non-movable line is called the directrix.

A parabola can be represented by the equation y² = – x.

The coordinates of the focus and the equation of the directrix will be given as

We know that the standard equation of the parabola y² = – 4ax.

On comparing, we have

a = 1/4 = 0.25

The focus of the parabola is at (–a, 0) that is (–0.25, 0).

The directrix of the parabola will be given as

y = a

x = 0.25

More about the parabola link is given below.

https://brainly.com/question/8495504

Answer:

A. focus: (negative one-fourth, 0); directrix: x = One-fourth

Step-by-step explanation:

i did it

Can someone help this is due in like 20 minutes Find the value of x in the triangle shown below. 58° 6 6 5.8 ​

Answers

Answer:

I'm wonde what it is because I like this stuff

The required value of angle x in the given triangle using the sine formula is 61.31

The triangle is shown in the figure, we have to determine the value of angle x in degrees.

To determine the angle, we must apply the sine formula using the angle-side combination.
Sine formula is given as:
sinA/a = sinB/b=sinC/c
sin58/5.8=sinx/6

Simplifying the above expression:
sinx=0.877
x = sin⁻¹(0.877)
x= 61.31°

Learn more about the sine formula here:

https://brainly.com/question/32029921

#SPJ4

the triangles below are congruent and the corresponding parts are marked

Answers

Answer:

Step-by-step explanation:

A yogurt machine makes
gallon of yogurt
every minute. The total amount y that the
machine makes in x minutes is given by
the equation y
]×. Determine whether the
relationship between x and y is linear and, if
so, if it is proportional,

Answers

Step-by-step explanation:

the relationship between x and y is defined by a constant factor.

you can "go" from x to y and from y to x consistently and without any interruptions by applying that constant factor.

that makes it a linear relationship (a line on a chart, hence the name).

proportional relationships are relationships between two variables where their ratios are equivalent. another way to think about them is that, in a proportional relationship, one variable is always a constant value times the other. That constant is know as the "constant of proportionality".

so, yes, it is proportional.

and the table is then

x y

0 0

3 3×2/3 = 2

6 6×2/3 = 4

9 9×2/3 = 6

A kite string is 100 feet long from the kite to the ground. The string makes a 45° angle with the ground
About how high off the ground is the kite? Leave your answer as a simplified radical

Answers

Answer:

It's not very high

Step-by-step explanation:

The angle that the string and the ground maje, is acute, because it's smaller than a right angle. So, since it is an acute angle, it's not very high. It is about 30cm high.

HELP PLEASE!!!!!!!!!

James left a bin outside in his garden to collect rainwater. He notices that 1/4 gallon of water fills 2/3 of the bin. Write and solve an expression to find the amount of water that will fill the entire bin. Show your work. Explain your answer in words

Answers

Answer:

3 pints of water will fill up the bin

Step-by-step explanation:

1 gallon = 4 quarts

1/4 gallon = 1 quart

1 quart = 2 pints

That means that Two pints filled 2/3 of the bin meaning that one more pint will fill it up completely

What is the number halfway between 8516 and 8586?

Answers

Answer:

8551

Step-by-step explanation:

The midpoint of two numbers can be found by finding the mean of the two numbersMidpoint = 8516 + 8586 / 2We clearly see both numbers start with the digits '85' therefore their midpoint will also start with '85'The remaining two digits on those two numbers should be added and divided by 2⇒ 16 + 86 / 2 = 102/2 = 51Midpoint = 8551

Solve for all values of x by factoring.
x2 – 2x – 28 = -6x + 4
X
=
There are two answers

Answers

Answer:

{4,-8}

Step-by-step explanation:

x^2 -2x -28 = -6x +4

x^2+4x-32=0

(x-4)(x+8) =0

x=4 or -8

what is the measure of angle F????

Answers

Answer:

Angel F would equal about 50 degrees.
I used my protractor to measure the angle. My answer may be incorrect since I measuring it through a computer screen. (please read more before submitting your answer)

Another strategy:
(this one should be more accurate)
The sum of exterior angles in a polygon is always equal to 360 degrees. Therefore, for all equiangular polygons, the measure of one exterior angle is equal to 360 divided by the number of sides in the polygon.


The angles are given all equal 312 degrees
75 + 91 + 146 = 312 degrees



This means angel F equals:
360 degrees - 312 degrees = F degrees

F = 48 degrees



In conclusion, 48 and 50 are reasonable answers but the strategy used to find the answer 48 degrees makes more sense.




Step-by-step explanation:

Have a great rest of your day
#TheWizzer

Other Questions
A socially constructed racial classification system in which a person of mixed racial heritage is automatically categorized as a member of the less (or least) privileged group is called Someone, please help me with this.A new student in the computer lab needs your help to access and write an email. Tell the new student in Spanish the directions that you take to navigate writing an email in 3-4 steps. Use a prior knowledge of verbs and new vocabulary to write a list or a short paragraph response. For example, I enter my username and password and click on the school's web page. I write a message and send the email to my teacher. = 1 2 + (2 + tan 1 2 ) where and are constants. Show that (1 + ) y/dy=y Which type of sentence is this? During World War I, President Woodrow Wilson kept sheep on the White House lawn and raised money for the Red Cross through the sale of their wool. What qualities of socialism appealed to working-class people?. Videos Incorporated charges a membership fee of $4.99 plus $0.99 for each movie you watch. Videos Plus does not charge a membership fee but charges $2.99 per movie. Videos Plus is a better deal than Videos Incorporated. Which inequality represents this situation?What is the inequality symbol for the situation? Answer Chocies: >, In 2009 dollars, u. S. Gdp was $1057 billion at the start of the great depression but fell to $778 billion by 1933. What percentage decline does this represent?. Graph the inequality of x>15 Which function should be used when evaluating the function at x = -1?S -x2 + 8x if x < -1p(x) =*x? - 4 if x 2-1O p (x) = {x? - 4O p(x) = -x2 + 8x (07.01 MC)A potential energy diagram is shown.200Potential Energy (kJ)150Reactants100Products500Reaction PathwayWhat is the activation energy of this reaction? (5 points)O 50 kJ100 kJO 150 kJO 200 kJ If the vertex of a parabola is (2,9), what is the axis of symmetry? I NEED THAT ANSWER ASAP What is the output for the following line of code?>>>int(2.8)O 3O '2.8022.8 In paragraph 1 of The Negro Artist and the Racial Mountain, how does Langston Hughes conclude that the young poet will never be a great poet?by suggesting that he believes the young poet is too individualisticby stating that the young poet is afraid to create lasting materialby tracing what he believes to be the young poets thought processby showing that there are too many barriers to the young poets success Arun and Benny started saving at the same time. Each day, Arunsaved $1.20 and Benny saved $0.30 more than Arun. How much wouldArun have saved if Benny saved $11.40 more than Arun? Jackson recorded the following data: 2,5, X, 2, 4, 3. If the meanof the data is 3, what is the value of x? Explain how you found your answer Jenna purchased 500 shares of XYZ stock for $10 per share. The stock paid the following dividends: Year 1: $0.25 per share Year 2: $0.27 per share Year 3: $0.29 per share Assume the stock is trading for $15 per share at the end of Year 3. Calculate the time-weighted return for XYZ stock over this period. The people in the North said that all people should be ______________. I NEED HELP PLEASE HELP ME What is the combination of a Protagonist and an Antagonist? Balancing chemical equation